How Geometry Solved the Most Difficult Number Theory Problem at the IMO 2001

แชร์
ฝัง
  • เผยแพร่เมื่อ 25 พ.ย. 2024

ความคิดเห็น • 13

  • @yashrawat3174
    @yashrawat3174 11 หลายเดือนก่อน +4

    A Proof for Strong version of Ptolemy - You can use law of Cosines in ABC and ADC and use the fact that opposite angles in a cyclic quadrilateral sum to 180
    Therefore you can eliminate cos terms by manipulations and then factor out the numerator to get the same expression as shown in video

  • @judo-rob5197
    @judo-rob5197 ปีที่แล้ว +12

    To your knowledge were there non geometric solutions to the problem?

    • @EulersBasement
      @EulersBasement  ปีที่แล้ว +10

      Yes, there are several number theoretical proofs to this problem. They are all rather technical, however. You can check some of them out here artofproblemsolving.com/community/c6h17474p119217

  • @JadeVanadiumResearch
    @JadeVanadiumResearch ปีที่แล้ว +7

    7:37 It's not clear to me how we know that ab+cd>ac+bd>ad+bc?

    • @EulersBasement
      @EulersBasement  ปีที่แล้ว +7

      ab+cd>ac+bd ab-ac>bd-cd a(b-c)>d(b-c) since b-c>0 this is equivalent to a>d which obviously holds. Similarly for the other one we get ac+bd>ad+bc a(c-d)>b(c-d) a>b.
      You may want to read about something called "Rearrangement inequality" as it describes the general theory for these kinds of inequalities.

    • @zihaoooi787
      @zihaoooi787 ปีที่แล้ว

      > which obviously holds
      I’m lost.
      Edit: never mind I substituted (c-d) as s and it made more sense for a sec

    • @anbreibiggamer8580
      @anbreibiggamer8580 9 หลายเดือนก่อน +1

      Not really related to the problem ,but can you prove that ab+bc+cd+da>=2ac+2bd?
      Edit:if a>b>c>d

  • @ashishjoel2102
    @ashishjoel2102 2 หลายเดือนก่อน

    But since alpha and beta are 60. xyw and zyw are equilateral triangles. So a=c and b=d which contradicts the inequality that a>b>c>d.

  • @semimaths
    @semimaths 3 หลายเดือนก่อน

    Thanks for the nice explanation. One question though, how did you claim WY is an integer just because b and d are integers?

    • @JorgePabloPorrasAlvarado
      @JorgePabloPorrasAlvarado 14 ชั่วโมงที่ผ่านมา

      Because he was refering to WY² since WY is the squared root of an integer, then WY² it's simply an integer.

  • @did3338
    @did3338 3 หลายเดือนก่อน

    Euclidean geometry (Evan chen) solved this by geometry

  • @glgou4647
    @glgou4647 ปีที่แล้ว

    i think i missed out somewhere... how is the strong version of the formula derived?

    • @yashrawat3174
      @yashrawat3174 11 หลายเดือนก่อน +2

      You can use law of Cosines in ABC and ADC and use the fact that opposite angles in a cyclic quadrilateral sum to 180
      Therefore you can eliminate cos terms by manipulations and then factor out the numerator to get the same expression